¡Los impulsos no son unitarios!

A diferencia de las rotaciones, las transformaciones de impulso no son unitarias. Por lo tanto, los generadores de refuerzo no son hermitianos. Cuando los impulsos inducen transformaciones en el espacio de Hilbert, ¿serán esas transformaciones unitarias? Creo que no. Si ese es el caso, ¿cuál es el significado físico de tales transformaciones no unitarias correspondientes a impulsos en el espacio de Hilbert?

Está relacionado, pero esta pregunta es sobre el caso relativista, pero el debate anterior, tanto la pregunta como la respuesta, se trata completamente del caso galileano no relativista, por lo que no es exactamente el mismo problema.
EStimado @Roopam. He estado siguiendo sus preguntas de sondeo con interés. Como muestra la respuesta de Luboš en los detalles de su específico, esta pregunta se responde recordando que la imagen de un grupo bajo una representación particular es, en general, diferente del grupo mismo. Como sabe por su otra pregunta, la imagen de O ( 1 , 3 ) bajo la representación adjunta A d es O ( 1 , 3 ) sí mismo. Pero esto no es cierto para un homomorfismo general. Ni de hecho es cierto en general incluso de A d sí mismo. Marcaste la pregunta como "teoría de la representación", por lo que parece que eres...
... aprendiendo a entender estas ideas. Solo necesitas seguir aprendiendo sobre las representaciones y sus propiedades. Cuando algo sufre una transformación espaciotemporal por un miembro de O ( 1 , 3 ) , el estado cuántico correspondiente debe sufrir una transformación unitaria (¡después de todo, el objeto debe terminar en algún estado!). Cuando esté listo, vea la referencia de Woit vinculada en mi respuesta aquí o también esta excelente referencia del inimitable John Baez arxiv.org/abs/0904.1556 también fue de gran ayuda para mí.
... También el teorema de Stone-von Neumann, el teorema de Wigner y la clasificación de Wigner son conceptos importantes en los que se está metiendo.

Respuestas (1)

En el espacio de Hilbert real de un sistema mecánico cuántico relativista consistente, las transformaciones de Lorentz, incluidos los impulsos, en realidad son unitarias, lo que también significa que los generadores j 0 i son tan hermíticos como los generadores de rotaciones j i j .

Decimos que el espacio de Hilbert forma una representación unitaria del grupo de Lorentz.

Lo que debe confundir al OP es el hecho de que la representación vectorial ordinaria compuesta de vectores ( t , X , y , z ) no es una representación unitaria de S O ( 3 , 1 ) . los S O ( 3 , 1 ) Las transformaciones no conservan ninguna invariante cuadrática positivamente definida construida a partir de las coordenadas. ( t , X , y , z ) . Después de todo, sabemos que una forma indefinida, t 2 X 2 y 2 z 2 , se conserva por las transformaciones de Lorentz. Entonces, en una representación como el espacio vectorial de tal ( t , X , y , z ) , los generadores j 0 i terminaría siendo anti-hermitiano en lugar de hermitiano.

Pero si toma una teoría invariante de Lorentz con un espacio de Hilbert definido positivo, como QED, la fórmula para j 0 i hace manifiesto que es un operador hermitiano, lo que significa que ψ | ψ se conserva por los impulsos de Lorentz! Las amplitudes de probabilidad complejas para diferentes estados C i comportarse de manera diferente a las coordenadas t , X , y , z arriba.

Tenga en cuenta que las transformaciones unitarias (no triviales) de S O ( 3 , 1 ) son inevitablemente de dimensión infinita. Las repeticiones de dimensión finita se pueden construir a partir de la representación vectorial fundamental anterior y son tan no unitarias como la representación vectorial. Pero eso no es cierto para las repeticiones de dimensión infinita. Por ejemplo, el espacio de estados de una partícula escalar en una QFT es una representación unitaria del grupo de Lorentz. Para cada pags m obedeciendo pags m pags m = metro 2 , y hay infinitamente (continuamente) muchos valores de dicho vector (en la capa de masa), la representación contiene un vector base (que se normalizan a la función delta de Dirac). Los impulsos simplemente los "permutan" a lo largo de la capa de masa, lo que hace obvio que la forma positivamente definida se conserva cuando se normaliza correctamente.

¿A qué fórmula te refieres con la fórmula para j 0 i ? Estoy leyendo a Weinberg y en su exposición (pág. 59) simplemente asume la j m v son hermíticos, por lo que U es unitario. Me gustaría saber si se puede formular de otra manera.
Estimado @usuario27084, j 0 i es esquemáticamente X 0 pags i X i pags 0 pero en teoría de campo, pags m tiene que escribirse como una integral de la densidad de energía o la densidad de cantidad de movimiento en el espacio. Déjeme suponer que sabe cómo escribir el momento total o la energía total como una integral que involucra todos los campos. Entonces, para obtener el generador de impulso, simplemente escriba las mismas integrales que definen pags 0 o pags i y agrega el X 0 o X i factor extra dentro de la integral - para obtener ambos términos en la diferencia con la que comencé.
Para ser más precisos, la fórmula que obtuviste hasta ahora es solo la parte orbital de j 0 i , la L 0 i , si lo desea. Para obtener el generador completo del impulso, también debe agregar los términos responsables del giro, el momento angular intrínseco, de todos los campos. Todos estos términos adicionales son expresiones bilineales simples en el campo cuya estructura de índice está completamente determinada por la covarianza de Lorentz. Aproximadamente la mitad de los libros de QFT contienen fórmulas explícitas para la forma de j m v para cualquier teoría de campo que discutan.
Tenga en cuenta que todos los términos son algunas integrales y los integrandos contienen factores como X 0 , X i , densidad de energía o densidad de cantidad de movimiento. Y luego el giro. Si observa todos estos factores, nunca hay una unidad imaginaria, etc., lo que hace casi manifiesto que todos los generadores, independientemente de que los índices sean espaciales o temporales, son hermitianos.
Hola profesor. ¿Por qué la cuantización canónica de las cargas de Noether asociadas con la transformación de Lorentz del fotón no es hermitiana? Clásicamente, estas cargas generan las transformaciones infinitesimales de Lorentz bajo el soporte de Dirac. Así que espero que bajo cuantización canónica deberían generar la representación unitaria de la transformación de Lorentz. Pero no son hermitianos.
¿Podría agregar alguna fórmula para mostrar lo que le parece no hermitiano? ¿Viste que mi respuesta argumentaba que los generadores completos correctos son hermitianos?